Límite clásico de la ecuación de Schrödinger

Existe un argumento bien conocido de que si escribimos la función de onda como ψ = A Exp ( i S / ) , dónde A y S son reales, y sustituimos esto en la ecuación de Schrödinger y tomamos el límite h 0 , entonces veremos que S satisface la ecuación de Hamilton-Jacobi (por ejemplo, consulte http://physics.bu.edu/~rebbi/hamilton_jacobi.pdf ).

Entiendo esto, sin embargo, siento que no entiendo la afirmación de que esto muestra que la mecánica cuántica se reduce a la mecánica clásica en el 0 límite. Estoy confundido porque pensaría que para mostrar que QM se reduce a CM tendríamos que demostrar que como 0 , | ψ ( X , t ) | 2 se vuelve muy estrecho y que su centro se mueve en una trayectoria clásica, es decir | ψ ( X , t ) | 2 = d ( X X clásico ( t ) ) . Y parece que el argumento anterior no muestra esto en absoluto. De hecho, tengo entendido que todo lo que importa para la medición física de la posición es | ψ | 2 (ya que esto da la distribución de probabilidad) y por lo tanto el factor de fase Exp ( i S / h ) parece no importar en absoluto.

Además, algunos libros (ver página 282 de http://www.scribd.com/doc/37204824/Introduction-to-Quantum-Mechanics-Schrodinger-Equation-and-Path-Integral-Tqw-darksiderg#download o páginas 50- 52 de Landau y Lifshitz) dan un argumento adicional al mencionado anteriormente. Dicen además que si ψ = A Exp ( i S / h ) , entonces | ψ | 2 = A 2 satisface la ecuación de continuidad clásica para un fluido con una velocidad correspondiente d S / d t , que en el h 0 límite es igual a la velocidad clásica.

Este argumento tiene más sentido para mí. Sin embargo, todavía tengo algunas preguntas sobre esto. (1) Sé que hay estados estacionarios cuyo módulo al cuadrado no evoluciona en el tiempo, lo que parece contradecir esta interpretación de un fluido que fluye con velocidad v. (2) La interpretación fluida parece sugerirme quizás que la función de onda se reduce en el límite clásico más a una onda que a una partícula. (3) Esto no muestra que la función de onda sea estrecha.

dab, considera aprender L A T mi X , consulte meta.math.stackexchange.com/questions/1773/… y en.wikipedia.org/wiki/…
Esto podría ayudar.
Relacionado: physics.stackexchange.com/q/17651/2451 y enlaces allí.
Su pregunta se estudia y responde en mi artículo reciente >U.Klein, What is the limit 0 de la teoría cuántica?, Am.J.Phys. vol.80, 1009 (2012). La versión preliminar está disponible aquí .

Respuestas (3)

La sutileza es que una función de onda arbitraria no se reduce a un punto del espacio de fase clásico en el límite 0 (pensar en el espacio de fase tiene más sentido ya que en el límite clásico uno debería tener coordenadas y momentos definidos).

Entonces uno podría preguntarse qué funciones de onda hacen. Y la respuesta es que el límite clásico se basa en los llamados estados coherentes, los estados que minimizan la relación de incertidumbre (aunque no conozco ningún teorema matemático que demuestre que siempre es cierto en el caso general, pero en todos los casos conocidos). ejemplos es de hecho así). Los estados cercanos a los coherentes pueden considerarse como una especie de "borrosidad cuántica", correspondiente a las correcciones cuasiclásicas de órdenes superiores en .

Se puede encontrar un ejemplo de esto para el oscilador armónico en Landau Lifshits.

En cuanto al argumento fluido. Acerca de su comentario (1): el | ψ | 2 porque el estado estacionario es de hecho estacionario, pero aún satisface la ecuación de continuidad ya que la corriente es cero para tales estados. Tus comentarios (2) y (3) tienen toda la razón porque, como ya dije, el límite clásico no se puede tomar sensatamente para estados arbitrarios, se construye a partir de estados coherentes.

Y también debo admitir que el argumento fluido dado de hecho no proporciona ninguna manifestación de límite clásico. Es solo una ilustración de que "todo se comporta razonablemente bien" para convencer a los lectores de que todo está bien y, presumiblemente, para desviar su atención del punto difícil y sutil: a menudo sucede en pag h y s i C s libros, probablemente sin querer :). El problema de una buena descripción del límite clásico es en realidad abierto (aunque a menudo subestimado), lo que lleva a preguntas bastante profundas, como la forma sistemática de obtener la geometría simpléctica del límite clásico. En mi opinión, también está relacionado con el problema de la reducción cuántica (conocido también como el "colapso de la función de onda").

Buena discusión. Según el comentario de U. Klein sobre el 0 límite de la teoría cuántica no es lo que la gente podría pensar. Es natural e intuitivo, como se explicó anteriormente, suponer que el límite clásico es una propiedad de cierta clase de estados.

Da la casualidad de que esa opinión es incorrecta. De hecho, puede obtener una recuperación exacta de la mecánica de puntos clásica hamiltoniana para cualquier valor de usando una ecuación de onda diferente:

i d d t | ψ = [ H ( q , pag ) 1 ^ + H q ( q , pag ) ( q ^ q ) + H pag ( q , pag ) ( pag ^ pag ) ] | ψ
Esta ecuación de onda propaga cualquier estado a lo largo de trayectorias clásicas y es única (es decir, puede derivarla y demostrar que es única). La no linealidad es esencial y proviene de la presencia de los valores esperados en los parámetros que multiplican cada uno de los tres términos del operador.

Cuando uno se familiariza con este resultado, queda claro por qué hay tanta confusión.

La gente pensó que el límite clásico debería estar contenido dentro de la teoría cuántica.

La simple verdad es que no lo es. El límite no existe dentro de la teoría.

Se trata de una ecuación muy específica que está fuera de la mecánica cuántica lineal. Sin embargo, es simple, ya que esta ecuación produce las relaciones de Ehrenfest esperadas:

d q d t = + H pag ( q , pag )
y
d pag d t = H q ( q , pag )
que es el límite clásico de los paquetes de ondas que siguen caminos clásicos.

La ecuación relevante se obtuvo por primera vez en KRW Jones (1991), "La ecuación clásica de Schroedinger" UM-P-91/45 (CSE) http://arxiv.org/abs/1212.6786 . En 1992 se publicó una versión más simple: KRW Jones (1992), "La mecánica clásica como ejemplo de mecánica cuántica generalizada" Phys. Rev. D45, R2590-R2594. http://link.aps.org/doi/10.1103/PhysRevD.45.R2590 . La preimpresión original que contiene la derivación y la prueba de unicidad se publicó en arXiv hace unos días (ver enlace arriba).

El hecho de que la ecuación no sea lineal puede explicar por qué la literatura la pasó por alto durante tanto tiempo. Es simple, pero no es trivial, ni es obvio hasta que lo sabes.

La derivación del CSE implica la teoría de grupos y una representación no lineal inusual del grupo de Heisenberg-Weyl. El sistema matemático de teoría cuántica no lineal involucrado es el descubierto primero por Weinberg e (independientemente) por Jones.

Una receta general de cómo tomar un límite clásico usando un parámetro adimensional λ 0 se da en el artículo: KRW Jones (1993), "Un método general para deformar la dinámica cuántica en dinámica clásica mientras se mantiene fijo" Phys. Rev. A48, 822-825.

Allí encontrará el argumento general de cómo hacer un límite clásico consistentemente para que se recuperen el espacio de fase, las trayectorias y todas las demás propiedades.

Es sorprendente que la comunidad de físicos aún no se haya dado cuenta de esto. Sin embargo, la pregunta es excelente y sutil, ya que las matemáticas involucradas no existían antes de 1989. La conexión entre esta área y la integral de trayectoria de Feynman es particularmente interesante.

Estimado Kingsley Jones: Para su información, Physics.SE tiene la política de que está bien citarse a sí mismo, pero debe indicarse clara y explícitamente en la respuesta misma, no en los enlaces adjuntos.
Lo siento, agregué la ecuación real y la expliqué más completamente de manera independiente.

El resultado principal del artículo "¿Cuál es el límite ℏ→0 de la teoría cuántica?" es que el límite clásico de la teoría cuántica no es la mecánica clásica sino una teoría estadística clásica. Mi "artículo técnico" ha sido escrito con la idea en mente de contribuir a la comprensión (la interpretación) de la teoría cuántica. La conclusión final del artículo presenta, en mi opinión, un fuerte argumento a favor de la interpretación estadística de la teoría cuántica. La motivación y las conclusiones se analizan con más detalle en las secciones I y VIII del documento. Estaré encantado de responder preguntas específicas, pero tenga en cuenta que casi todas las respuestas que puedo dar se pueden encontrar en mi sitio web "http://statintquant.net" (solo estoy actualizando esto, terminaré pronto).

He fusionado sus cuentas no registradas nuevamente.
-1 por no explicar nada. ¿Y por qué estás publicando una nueva respuesta en lugar de editar tu publicación anterior?